Đến nội dung

Hình ảnh

VMF's Marathon Bất Đẳng Thức Olympic

marathon aops vmf

  • Please log in to reply
Chủ đề này có 162 trả lời

#21
Gachdptrai12

Gachdptrai12

    Thượng sĩ

  • Điều hành viên THCS
  • 280 Bài viết

Bài toán 8. Cho $a,b,c$ là các số thực dương. Chứng minh

\[\frac{\sqrt{a+b}}{c}+\frac{\sqrt{b+c}}{a}+\frac{\sqrt{a+c}}{b}\geq 3\sqrt{\frac{2(a+b+c)}{ab+bc+ca}}\]


Bài viết đã được chỉnh sửa nội dung bởi hoanglong2k: 25-05-2016 - 16:40


#22
tuanyeubeo2000

tuanyeubeo2000

    Hạ sĩ

  • Thành viên
  • 80 Bài viết

 

Bài toán 8. Cho $a,b,c$ là các số thực dương. Chứng minh
\[\frac{\sqrt{a+b}}{c}+\frac{\sqrt{b+c}}{a}+\frac{\sqrt{a+c}}{b}\geq 3\sqrt{\frac{2(a+b+c)}{ab+bc+ca}}\]

Lời giải bài 8. Áp dụng AM-GM ta có $\sum \dfrac{\sqrt{a+b}}{c}\geq 3\sqrt[3]{\sqrt{\dfrac{(a+b)(b+c)(c+a)}{a^2b^2c^2}}}$
Và $(a+b)(b+c)(c+a)\geq \dfrac{8}{9}(a+b+c)(ab+bc+ca)$
Ta sẽ chứng minh $\sqrt[3]{\sqrt{\dfrac{8(a+b+c)(ab+bc+ca)}{9a^2b^2c^2}}}\geq \sqrt{\dfrac{2(a+b+c)}{ab+bc+ca}}$ (*)
Đặt $a+b+c=p; ab+bc+ca=q ; abc=r$ Thì (*)$\Leftrightarrow q^2\geq 3pr$ đúng
Vậy bất đẳng thức được chứng minh. Dấu "=" xảy ra khi $a=b=c$

 

Bài toán 9. Cho $a,b,c>0$. Chứng minh rằng
\[\sqrt{\dfrac{a+b}{c}}+\sqrt{\dfrac{b+c}{a}}+\sqrt{\dfrac{c+a}{b}}\geq 2\left(\sqrt{\dfrac{a}{b+c}}+\sqrt{\dfrac{b}{c+a}}+\sqrt{\dfrac{c}{a+b}}\right) \]


Bài viết đã được chỉnh sửa nội dung bởi hoanglong2k: 25-05-2016 - 18:00

Hiện tại là tặng phẩm vì theo cách chơi chữ trong tiếng anh thì hai từ nãy gần như là một 

Nên người nước ngoài luôn đưa ra một chân lý và chứng minh nó bằng ý nghĩa của họ chứ không phải cách tạo nên hai từ đó 

Vậy nên : Qùa tặng là cuộc sống hiện tại - Hãy nắm nó thật chắc


#23
Ngockhanh99k48

Ngockhanh99k48

    Trung sĩ

  • Thành viên
  • 127 Bài viết

Lời giải bài 9.
Ta có $\sum_{cyc}\sqrt{\dfrac{a+b}{2c}} \geq \sum_{cyc}\dfrac{\sqrt{a}+\sqrt{b}}{2\sqrt{c}} \geq \dfrac{1}{2}\sum_{cyc}\sqrt{a}\left (\frac{1}{\sqrt{b}}+\dfrac{1}{\sqrt{c}}\right ) \geq \sum_{cyc}\dfrac{2\sqrt{a}}{\sqrt{b}+\sqrt{c}} \geq \sum_{cyc}\sqrt{\dfrac{2a}{b+c}}$.

 

Bài toán 10. Cho $a, b, c >0$. Chứng minh rằng:
\[(a+b+c)\left(\dfrac{a}{b^2+c^2} + \dfrac{b}{c^2+a^2} + \dfrac{c}{a^2+b^2}\right) \geq 4+ \dfrac{4a^2b^2c^2}{(a^2+b^2)(b^2+c^2)(c^2+a^2)}\]

 

 \begin{array}{| l | l |} \hline \text{HDTterence2k} & 1\\ \hline \text{hoanglong2k} & 3\\ \hline \text{Gachdptrai12} & 4\\ \hline \text{Nguyenhuyen_AG} & 3\\ \hline \text{fatcat12345} & 1\\ \hline \text{lenhatsinh3} & 1\\ \hline \text{tuanyeubeo2000} & 1\\ \hline \text{Ngockhanh99k48} & 1 \\ \hline \end{array} 


Bài viết đã được chỉnh sửa nội dung bởi hoanglong2k: 25-05-2016 - 18:07


#24
fatcat12345

fatcat12345

    Binh nhất

  • Banned
  • 46 Bài viết

Lời giải bài 10.

$(*)\Leftrightarrow \sum S_a(b-c)^2\geq 0$

với

$S_a=(b^2+c^2+4bc-2a^2)(b^2+c^2)+2a(b+c)^2$

$S_b=(c^2+a^2+4ca-2b^2)(c^2+a^2)+2b(c+a)^2$

$S_c=(a^2+b^2+4ab-2c^2)(a^2+b^2)+2c(a+b)^2$

Giả sử $a\geq b\geq c$

Suy ra $S_c\geq S_b\geq S_a$

Và $S_a+S_b\geq 2c^2a^2-2b^2c^2\geq 0$

Nên $\sum S_a(b-c)^2\geq (S_c+S_b)(a-b)^2+(S_b+S_a)(b-c)^2\geq 0\blacksquare$ 

 

\begin{array}{| l | l |} \hline \text{HDTterence2k} & 1\\ \hline \text{hoanglong2k} & 3\\ \hline \text{Gachdptrai12} & 4\\ \hline \text{Nguyenhuyen_AG} & 3\\ \hline \text{fatcat12345} & 2\\ \hline \text{lenhatsinh3} & 1\\ \hline \text{tuanyeubeo2000} & 1\\ \hline \text{Ngockhanh99k48} & 1 \\ \hline \end{array} 


Bài viết đã được chỉnh sửa nội dung bởi hoanglong2k: 25-05-2016 - 20:20


#25
MathematicsNMN2016

MathematicsNMN2016

    Trung sĩ

  • Thành viên
  • 126 Bài viết

 Mình xin được đóng góp 1 bài toán thú vị sau:

 
 Bài toán 11:
 Cho $x, y, z$ và $t$ là bốn số thực dương, thoả mãn:
$\left ( x+y+z+t \right )\left ( \frac{1}{x}+\frac{1}{y}+\frac{1}{z}+\frac{1}{t} \right )=20.$
 Chứng minh rằng:
$36\leq \left ( x^{2}+y^{2}+z^{2}+t^{2} \right )\left ( \frac{1}{x^{2}}+\frac{1}{y^{2}}+\frac{1}{z^{2}}+\frac{1}{t^{2}} \right )\leq 580-240\sqrt{5}.$

Bài viết đã được chỉnh sửa nội dung bởi hoanglong2k: 25-05-2016 - 20:22


#26
tuanyeubeo2000

tuanyeubeo2000

    Hạ sĩ

  • Thành viên
  • 80 Bài viết

Lời giải bài 9.
Ta có $\sum_{cyc}\sqrt{\dfrac{a+b}{2c}} \geq \sum_{cyc}\dfrac{\sqrt{a}+\sqrt{b}}{2\sqrt{c}} \geq \dfrac{1}{2}\sum_{cyc}\sqrt{a}\left (\frac{1}{\sqrt{b}}+\dfrac{1}{\sqrt{c}}\right ) \geq \sum_{cyc}\dfrac{2\sqrt{a}}{\sqrt{b}+\sqrt{c}} \geq \sum_{cyc}\sqrt{\dfrac{2a}{b+c}}$.

Một cách khác cho bài 9 :

$\text{BĐT}\Leftrightarrow \sum \left(\sqrt{\frac{b+c}{a}}-2\sqrt{\frac{a}{b+c}}\right)\ge 0\Leftrightarrow \sum \frac { b+c-2a }{ \sqrt { a(b+c) }  }\ge 0\ \ (*)$

Không mất tính tổng quát, giả sử $a\geq b\geq c$ thì $\begin{cases} b+c-2a\le c+a-2b\le a+b-2c \\ \frac { 1 }{ \sqrt { a(b+c) }  } \le \frac { 1 }{ \sqrt { b(c+a) }  } \le \frac { 1 }{ \sqrt { c(a+b) }  }  \end{cases}$

Đến đây áp dụng bất đẳng thức Chebyshev ta có $VT(*)\ge (2a+2b+2c-2a-2b-2c)\left( \frac { 1 }{ \sqrt { a(b+c) }  } +\frac { 1 }{ \sqrt { b(c+a) }  } +\frac { 1 }{ \sqrt { c(a+b) }  }  \right) =0$

Vậy ta có điều cần chứng minh


Bài viết đã được chỉnh sửa nội dung bởi hoanglong2k: 26-05-2016 - 22:21

Hiện tại là tặng phẩm vì theo cách chơi chữ trong tiếng anh thì hai từ nãy gần như là một 

Nên người nước ngoài luôn đưa ra một chân lý và chứng minh nó bằng ý nghĩa của họ chứ không phải cách tạo nên hai từ đó 

Vậy nên : Qùa tặng là cuộc sống hiện tại - Hãy nắm nó thật chắc


#27
Nguyenhuyen_AG

Nguyenhuyen_AG

    Trung úy

  • Thành viên nổi bật 2016
  • 945 Bài viết

Bài toán 10. Cho $a, b, c >0$. Chứng minh rằng:
\[(a+b+c)\left(\dfrac{a}{b^2+c^2} + \dfrac{b}{c^2+a^2} + \dfrac{c}{a^2+b^2}\right) \geq 4+ \dfrac{4a^2b^2c^2}{(a^2+b^2)(b^2+c^2)(c^2+a^2)}. \quad (1)\]

 

Lời giải bài 10. Bất đẳng thức $(1)$ tương đương với

\[(a+b+c)\left [\sum a(a^2+b^2)(a^2+c^2)  \right ] \geqslant 4(a^2+b^2)(b^2+c^2)(c^2+a^2)+4a^2b^2c^2.\]

Chuyển về pqr như sau

\[p\left [(3p^2-5q)r+p(p^2-3q)(p^2-2q) \right ] \geqslant 4(p^2-2q)(q^2-2pr),\]

thu gọn thành

\[p(11p^2-21q)r+(p^2-4q)(p^2-2q)(p^2+q) \geqslant 0. \quad (2)\]

Chú ý rằng $p^2 \geqslant 3q \geqslant 2q$ cho nên

  • Nếu $p^2 \geqslant 4q$ thì $(2)$ đúng.
  • Nếu $p^2 \leqslant 4q$ theo bất đẳng thức Schur bậc $4$ ta có $r \geqslant \frac{(4q-p^2)(p^2-q)}{6p}$ áp dụng điều này ta đưa bài toán về \[(11p^2-21q) \cdot \frac{(4q-p^2)(p^2-q)}{6}+(p^2-4q)(p^2-2q)(p^2+q) \geqslant 0,\] thu gọn thành \[\frac{1}{6}(5p^2-11q)(p^2-3q)(4q-p^2) \geqslant 0.\] Hiển nhiên đúng trong trường hợp mà ta đang xem xét.

Ta có điều phải chứng minh.


Bài viết đã được chỉnh sửa nội dung bởi Nguyenhuyen_AG: 27-05-2016 - 18:11

Nguyen Van Huyen
Ho Chi Minh City University Of Transport

#28
tuanyeubeo2000

tuanyeubeo2000

    Hạ sĩ

  • Thành viên
  • 80 Bài viết

 

Mình xin được đóng góp 1 bài toán thú vị sau:

 

Bài toán 11:
Cho $x, y, z$ và $t$ là bốn số thực dương, thoả mãn:

$\left ( x+y+z+t \right )\left ( \frac{1}{x}+\frac{1}{y}+\frac{1}{z}+\frac{1}{t} \right )=20.$

Chứng minh rằng:

$36\leq \left ( x^{2}+y^{2}+z^{2}+t^{2} \right )\left ( \frac{1}{x^{2}}+\frac{1}{y^{2}}+\frac{1}{z^{2}}+\frac{1}{t^{2}} \right )\leq 580-240\sqrt{5}.$

Lời giải bài 11. (Bài giải bởi bạn Triển - FB : Dinh de Tai)
Từ điều kiện ta có $\sum \dfrac{x+y+z}{t}=16$
Trước hết ta chứng minh:$\left(x^2+y^2+z^2+t^2\right)\left(\frac{1}{x^2}+\frac{1}{y^2}+\frac{1}{z^2}+\frac{1}{t^2}\right)\ge 36 \ \ \ \ (1)$
Chú ý $4(x^2+y^2+z^2+t^2)=(x+y+z-t)^2+(y+z+t-x)^2+(z+t+x-y)^2+(t+x+y-z)^2\Rightarrow \text{VT(1)}=\dfrac{1}{4}\sum (y+z+t-x)^2.\sum \dfrac{1}{x^2}$
Áp dụng bất đẳng thức Cauchy Schwarz ta có $\sum (y+z+t-x)^2.\sum \dfrac{1}{x^2}\geq \left[\sum \dfrac{y+z+t-x}{x}\right]^2=\left[\sum \dfrac{y+z+t+x}{x}-8\right]^2=144$
Suy ra $\text{VT(1)}\geq \dfrac{1}{4}.144=36$
Bây giờ ta sẽ chứng minh: $\left(x^2+y^2+z^2+t^2\right)\left(\frac{1}{x^2}+\frac{1}{y^2}+\frac{1}{z^2}+\frac{1}{t^2}\right)\le 580-240\sqrt { 5 }$
Áp dụng bất đẳng thức Cauchy Schwarz ta có $\sqrt{\sum x^2.\sum \dfrac{1}{x^2}}+\sqrt{2\sum_{sym}xy.2\sum_{sym}\dfrac{1}{xy}}\leq \sqrt{\left(\sum x\right)^2\left(\sum \dfrac{1}{x}\right)^2}=20$
Nên ta chỉ cần chứng minh
$(xy+yz+zt+tx+xz+ty)\left(\dfrac{1}{xy}+\dfrac{1}{yz}+\dfrac{1}{zt}+\dfrac{1}{tx}+\dfrac{1}{xz}+\dfrac{1}{yt}\right)\geq 45$
$\Leftrightarrow 6+\sum_{sym} \dfrac{xy}{zt}+2\sum _{cyc} \dfrac{x+y+z}{t}\geq 45$
$\Leftrightarrow \sum_{sym} \dfrac{xy}{zt}\geq 7\Leftrightarrow \dfrac{(xy)^2+(yz)^2+(zt)^2+(tx)^2+(xz)^2+(ty)^2}{xyzt}\geq 7$
Áp dụng bất đẳng thức Cauchy-Schwarz ta có $(xy)^2+(yz)^2+(zt)^2+(tx)^2+(xz)^2+(ty)^2=x^2(y^2+z^2+t^2)+(yz)^2+(zt)^2+(ty)^2\geq x^2(yz+zt+ty)+yzt(y+z+t)$
Suy ra $\dfrac{(xy)^2+(yz)^2+(zt)^2+(tx)^2+(xz)^2+(ty)^2}{xyzt}\geq \dfrac{x}{y}+\dfrac{x}{z}+\dfrac{x}{t}+\dfrac{y+z+t}{x}$
Tương tự thì $4\dfrac{(xy)^2+(yz)^2+(zt)^2+(tx)^2+(xz)^2+(ty)^2}{xyzt}\geq \sum x\left(\dfrac{1}{y}+\dfrac{1}{z}+\dfrac{1}{t}\right)+16=32$
$\Rightarrow \dfrac{(xy)^2+(yz)^2+(zt)^2+(tx)^2+(xz)^2+(ty)^2}{xyzt}\geq 8>7$
Do đó ta có điều cần chứng minh

 

 @hoanglong2k : Mình không cộng điểm cho Tuấn nhưng sẽ cộng điểm cho Triển 


Bài viết đã được chỉnh sửa nội dung bởi hoanglong2k: 29-05-2016 - 09:24

Hiện tại là tặng phẩm vì theo cách chơi chữ trong tiếng anh thì hai từ nãy gần như là một 

Nên người nước ngoài luôn đưa ra một chân lý và chứng minh nó bằng ý nghĩa của họ chứ không phải cách tạo nên hai từ đó 

Vậy nên : Qùa tặng là cuộc sống hiện tại - Hãy nắm nó thật chắc


#29
Gachdptrai12

Gachdptrai12

    Thượng sĩ

  • Điều hành viên THCS
  • 280 Bài viết

Bài toán 12. (Phan Hồng Sơn) Cho $a,b,c>0$ thỏa $a+b+c=3$. Chứng minh rằng

$\frac{a^{2}}{\sqrt{bc+(a-1)^{2}}}+\frac{b^{2}}{\sqrt{ca+(b-1)^{2}}}+\frac{c^{2}}{\sqrt{ab+(c-1)^{2}}}\geq 3$


Bài viết đã được chỉnh sửa nội dung bởi hoanglong2k: 29-05-2016 - 09:25


#30
fatcat12345

fatcat12345

    Binh nhất

  • Banned
  • 46 Bài viết

Lời giải bài 12.

Áp dụng bất đẳng thức $Cauchy-Schwarz$ ta có

\[\sum_{cyc}\frac{a^2}{\sqrt{bc+(a-1)^2}}=\sum_{cyc}\frac{a^4}{a^2\sqrt{bc+(a-1)^2}}\geq \frac{(\sum_{cyc}a^2)^2}{\sum_{cyc}a^2\sqrt{bc+(a-1)^2}}\geq \frac{(\sum_{cyc}a^2)^2}{\sqrt{(\sum_{cyc}a^3)\left \{ \sum_{cyc}\left [ a(a-1)^2+abc \right ] \right \}}}\]
nên ta chỉ cần chứng minh
\[(\sum_{cyc}a^2)^4\geq 9(\sum_{cyc}a^3)\left \{ \sum_{cyc}\left [ a(a-1)^2+abc \right ] \right \}\]
Đổi biến $pqr$
\[(9-2q)^4\geq 9(27-9q+3r)(12-5q+6r)\]
Để ý $9\geq q^2\geq 3pr=9r$ nên ta chỉ cần chứng minh
\[(9-2q)^4\geq (81-27q+q^2)(36-15q+2q^2)\]
tương đương với
\[(3-q)^2\left [ 14q^2+135(3-q) \right ]\geq 0\]
bất đẳng thức này hiển nhiên đúng $\blacksquare $
 
Bài toán 13. (Sưu tầm) Cho $a,b,c>0$ thỏa $a+b+c=1$. Chứng minh

\[\frac{a}{b+c}+\frac{b}{a+c}+\frac{c}{a+b} \geq \sqrt{\frac{4-27abc}{4(ab+bc+ac)}}\]


Bài viết đã được chỉnh sửa nội dung bởi hoanglong2k: 29-05-2016 - 09:25


#31
Gachdptrai12

Gachdptrai12

    Thượng sĩ

  • Điều hành viên THCS
  • 280 Bài viết

Bài toán 10. Cho $a, b, c >0$. Chứng minh rằng:
\[(a+b+c)\left(\dfrac{a}{b^2+c^2} + \dfrac{b}{c^2+a^2} + \dfrac{c}{a^2+b^2}\right) \geq 4+ \dfrac{4a^2b^2c^2}{(a^2+b^2)(b^2+c^2)(c^2+a^2)}\]

 Lời giải 3 cho bài 10:
 Viết bài toán dưới dạng $pqr$ là $p(11p^{2}-21q)r+(p^{2}-4q)(p^{2}-2q)(p^2+q)\geq0$
 Dễ thấy hệ số cuả $r$ là số dương nên theo phương pháp ABC thì cực trị đạt được tại 2 biến bằng nhau và một biến bằng $0$
 TH1: $b=c$ và do tính đồng bậc ta có thể giả sử $b=1$, do đó bất đẳng thức ban đầu trở thành
$(a+2)(\frac{a}{2}+\frac{2}{a^{2}+1}\geq 4+\frac{4a^2}{2(a^{2}+1)^{2}}$
$\Leftrightarrow a(a-1)^{2}(4a^{3}+4a^{2}+a+6)\geq0$ (hiển nhiên đúng)
 TH2: $c=0$ bất đẳng thức ban đầu trở thành
$(a+b)(\frac{a}{b^{2}}+\frac{b}{a^{2}})\geq4$
Tương đương $(a+b)(a^{3}+b^{3})\geq4a^{2}b^{2}$ (hiển nhiên đúng theo AM-GM)
 Vậy bất đẳng thức được chứng minh

 

PS


Bài viết đã được chỉnh sửa nội dung bởi Gachdptrai12: 04-08-2016 - 11:22


#32
Gachdptrai12

Gachdptrai12

    Thượng sĩ

  • Điều hành viên THCS
  • 280 Bài viết

Biến đổi theo $p,q,r$ ta được $: \frac{3r-2q+1}{q-r} \geq \sqrt{\frac{4-27r}{4q}} $
Áp dụng $: r \geq max \left \{ 0, \frac{4q-1}{9} \right \} (1)$
TH1 $: q \leq 0,25 \Rightarrow VT (1) \geq \frac{-2q+1}{q} $
Cần cm $: \frac{-2q+1}{q} \geq \sqrt{\frac{1}{q}} (2) $
Vì $q>0$ nên $: (2) \Leftrightarrow (q-1)(4q-1) \geq 0 (đúng ) $
TH2 $: q \geq 0,25 $
p\s $:$ Ai rảnh xét hộ e TH $2 $

 Bạn ơi bạn chưa xét đượ̣c hàm đó là đồng biến hay nghịch biến theo $r$ đâu mà cho vô ầm ầm thế


Bài viết đã được chỉnh sửa nội dung bởi hoanglong2k: 29-05-2016 - 09:28


#33
Hoang Tung 126

Hoang Tung 126

    Thiếu tá

  • Thành viên
  • 2061 Bài viết

 

 
Bài 13. (Sưu tầm) Cho $a,b,c>0$ thỏa $a+b+c=1$. Chứng minh

\[\frac{a}{b+c}+\frac{b}{a+c}+\frac{c}{a+b} \geq \sqrt{\frac{4-27abc}{4(ab+bc+ac)}}\]

 Bạn xem lại đề bài câu này nhé. Nghe chừng đề có vấn đề !



#34
fatcat12345

fatcat12345

    Binh nhất

  • Banned
  • 46 Bài viết

 Bạn xem lại đề bài câu này nhé. Nghe chừng đề có vấn đề !

Vấn đề gì vậy bạn? Mình không thấy có vấn đề gì cả  :closedeyes:



#35
Gachdptrai12

Gachdptrai12

    Thượng sĩ

  • Điều hành viên THCS
  • 280 Bài viết

Bài 13. (Sưu tầm) Cho $a,b,c>0$ thỏa $a+b+c=1$. Chứng minh

\[\frac{a}{b+c}+\frac{b}{a+c}+\frac{c}{a+b} \geq \sqrt{\frac{4-27abc}{4(ab+bc+ac)}}\]

Lời giải bài 13: Không mất tính tổng quát giả sử $a\geq b\geq c$, ta có một đẳng thức$(x+y+z)^{2}=3(xy+yz+zx)+\frac{1}{2}\sum (x-y)^{2}$

Từ đó ta có

BĐT$\Leftrightarrow (\frac{a}{b+c}+\frac{b}{c+a}+\frac{c}{a+b})^{2}\geq \frac{4(a+b+c)^{3}-27abc}{4(a+b+c)(ab+bc+ca)}$

Áp dụng đẳng thức trên ta có

$(\sum \frac{a}{b+c})^{2}=3\sum \frac{ab}{(a+c)(b+c)}+\frac{1}{2}\sum (\frac{a}{b+c}-\frac{b}{c+a})^{2}$

$=3\frac{\sum ab(a+b)}{(a+b)(b+c)(c+a)}+\frac{(a+b+c)^{2}}{2}\sum \frac{(a-b)^{2}}{(a+c)^{2}(b+c)^{2}}$

Mà $3 \frac{\sum ab(a+b)}{(a+b)(b+c)(c+a)}-\frac{9}{4}=\frac{3}{4}\sum \frac{c(a-b)^{2}}{(a+b)(b+c)(c+a)}$

Và $\frac{4(a+b+c)^{3}-27abc}{4(a+b+c)(ab+bc+ca)}-\frac{9}{4}=\frac{4(a^{3}+b^{3}+c^{3}-3abc)+3(a+b)(b+c)(c+a)-24abc}{4(a+b+c)(ab+bc+ca)}$

$= \sum \frac{(a-b)^{2}}{2(ab+bc+ca)}+\frac{3}{4}\sum \frac{c(a-b)^{2}}{(a+b+c)(ab+bc+ca)}\leq \sum \frac{(a-b)^{2}}{2(ab+bc+ca)}+\frac{3}{4}\sum \frac{c(a-b)^{2}}{(a+b)(b+c)(c+a)}$

Từ đó ta chỉ cần chứng minh được

$\sum (a-b)^{2}[\frac{(a+b+c)^{2}}{2(a+c)^{2}(b+c)^{2}}-\frac{1}{2(ab+bc+ca)}]\Leftrightarrow \sum S_{c}(a-b)^{2}\geq 0$

Dễ thấy $S_{c}\geq S_{b}\geq S_{a}$ ta có

$S_{a}+S_{b}=\frac{(a+b+c)^{2}}{2(a+b)^{2}(b+c^{2})}+\frac{(a+b+c)^{2}}{2(a+c)^{2}(b+c^{2})}-\frac{1}{ab+bc+ca}=\frac{(a+b+c)^{2}}{(a+b)^{2}(a+c)(b+c)}-\frac{1}{ab+bc+ca}\geq \frac{a+b+c}{(a+b)(b+c)(c+a)}-\frac{1}{ab+bc+ca}=\frac{abc}{(a+b+c)(ab+bc+ca)}\geq 0$

Từ đó ta có $S_{a}+S_{b}\geq 0$. Nếu $S_{a}\geq0$ thì bđt hiển nhiên đúng vì $S_{c}\geq S_{b}\geq S_{a} \geq 0$

Còn nếu $S_{a}\leq 0\Rightarrow S_{b}\geq 0$ (vì $S_{a}+S_{b}\geq 0$)

Ta chú ý tới bđt sau $S_{b}(a-c)^{2}\geq S_{b}(b-c)^{2}\Rightarrow S_{b}(a-c)^{2}+S_{a}(b-c)^{2}\geq (b-c)^{2}(S_{a}+S_{b})\geq 0$

Vậy ta có đpcm

 

Bài toán 14. (Micheal Rozenberg)

Cho $a,b,c$ dương thỏa $a+b+c=1$. Chứng minh

$\frac{a+b}{ab+1}+\frac{b+c}{bc+1}+\frac{c+a}{ca+1}\geq \frac{9}{5}$


Bài viết đã được chỉnh sửa nội dung bởi hoanglong2k: 29-05-2016 - 09:42


#36
fatcat12345

fatcat12345

    Binh nhất

  • Banned
  • 46 Bài viết

Lời giải bài 14

Bất đẳng thức ban đầu tương đương

$$\sum_{cyc}S_c(a-b)^2\geq 0$$

với

$$S_a=2bc-4ca-4ab+1-abc(a+3b+3c)$$
$$S_b=2ca-4ab-4bc+1-abc(3a+b+3c)$$
$$S_c=2ab-4bc-4ca+1-abc(a+3b+3c)$$
Giả sử $a\geq b\geq c$ suy ra $S_c\geq S_b\geq S_a$
Mà 
$$\frac{1}{2}(S_a+S_b)=a^2+b^2-2ab+c(a+b+c)-abc(2a+2b+3c)\geq c\left [ (a+b+c)^3-abc(2a+2c+3c) \right ]\geq 0$$
Nên
$$\sum_{cyc}S_c(a-b)^2\geq (S_c+S_b)(a-b)^2+(S_b+S_a)(b-c)^2\geq 0\blacksquare$$
 
Bài 15. Cho $a,b,c>0$ sao cho $ab+bc+ca=1$. Chứng minh
$$(a^2+ab+b^2)(b^2+bc+c^2)(c^2+ca+a^2)\ge\frac{2}{3}(\frac{a}{b+c}+\frac{b}{c+a}+\frac{c}{a+b}).$$

Bài viết đã được chỉnh sửa nội dung bởi fatcat12345: 28-05-2016 - 17:58


#37
Ngockhanh99k48

Ngockhanh99k48

    Trung sĩ

  • Thành viên
  • 127 Bài viết

 Một lời giải khác cho bài toán 14:
 Đặt $\frac{a+b}{2}=z, \frac{b+c}{2}=x, \frac{a+c}{2}=y$.

 Ta có $x+y+z=1$, sử dụng AM-GM ta có $\frac{a+b}{ab+1} \geq \frac{a+b}{(\frac{a+b}{2})^2+1}=\frac{2z}{z^2+1}$.

 Phần còn lại ta chứng minh $\frac{x}{x^2+1} +\frac{y}{y^2+1} +\frac{z}{z^2+1} \geq \frac{9}{10}$.

 Đó là BĐT Polish 96 nổi tiếng.


Bài viết đã được chỉnh sửa nội dung bởi hoanglong2k: 29-05-2016 - 09:31


#38
trungvmfcsp

trungvmfcsp

    Binh nhì

  • Thành viên mới
  • 13 Bài viết

Một lời giải mới bài 14. $\frac{a+b}{ab+1}+\frac{b+c}{bc+1}+\frac{c+a}{ca+1}\geq \frac{9}{5}$

Ta có: $\frac{a+b}{ab+1}+\frac{b+c}{bc+1}+\frac{c+a}{ca+1} =\frac{(a+b)^{2}}{(a+b)(ab+1)}+\frac{(b+c)^{2}}{(b+c)(bc+1)}+\frac{(c+a)^{2}}{(c+a)(ca+1)}$

$\geq \frac{(2(a+b+c))^{2}}{ab(a+b)+bc(b+c)+ca(c+a)+2(a+b+c)}=\frac{4}{2+ab(1-c)+bc(1-a)+ca(1-b)}= \frac{4}{2+ab+bc+ca-3abc}$

Nên ta chỉ cần chứng minh $\frac{4}{2+ab+bc+ca-3abc}\geq \frac{9}{5} \Leftrightarrow 2+27abc\geq 9(ab+bc+ca)$ (1)

Áp dụng bất đẳng thức Schur ta có:

$9abc\geq 4(ab+bc+ca)-1\Leftrightarrow 27abc>=12(ab+bc+ca)-3$

Thay $27abc\geq 12(ab+bc+ca)-3$ vào (1) ta có: $ab+bc+ca \leq \frac{1}{3}$

Bất đẳng thức đúng do $a + b + c = 1$, ta có đpcm.

Đẳng thức xảy ra khi $a = b = c = \frac{1}{3}$


Bài viết đã được chỉnh sửa nội dung bởi hoanglong2k: 29-05-2016 - 09:33


#39
fatcat12345

fatcat12345

    Binh nhất

  • Banned
  • 46 Bài viết

Một lời giải khác cho bài toán 14:
Đặt $\frac{a+b}{2}=z, \frac{b+c}{2}=x, \frac{a+c}{2}=y$. Ta có $x+y+z=1$, sử dụng AM-GM ta có $\frac{a+b}{ab+1} \geq \frac{a+b}{(\frac{a+b}{2})^2+1}=\frac{2z}{z^2+1}$. Phần còn lại ta chứng minh $\frac{x}{x^2+1} +\frac{y}{y^2+1} +\frac{z}{z^2+1} \geq \frac{9}{10}$. Đó là BĐT Polish 96 nổi tiếng.

Mình nhớ BĐT Poland 96 là $\frac{x}{x^2+1} +\frac{y}{y^2+1} +\frac{z}{z^2+1} \leq \frac{9}{10}$ chứ nhỉ  :D 



#40
Nguyenhuyen_AG

Nguyenhuyen_AG

    Trung úy

  • Thành viên nổi bật 2016
  • 945 Bài viết

Một lời giải khác cho bài toán 14:
Đặt $\frac{a+b}{2}=z, \frac{b+c}{2}=x, \frac{a+c}{2}=y$. Ta có $x+y+z=1$, sử dụng AM-GM ta có $\frac{a+b}{ab+1} \geq \frac{a+b}{(\frac{a+b}{2})^2+1}=\frac{2z}{z^2+1}$. Phần còn lại ta chứng minh $\frac{x}{x^2+1} +\frac{y}{y^2+1} +\frac{z}{z^2+1} \geq \frac{9}{10}$. Đó là BĐT Polish 96 nổi tiếng.

 

Bất đẳng thức trong đề thi của Ba Lan có chiều ngược lại.


Nguyen Van Huyen
Ho Chi Minh City University Of Transport





Được gắn nhãn với một hoặc nhiều trong số những từ khóa sau: marathon, aops, vmf

3 người đang xem chủ đề

0 thành viên, 3 khách, 0 thành viên ẩn danh